PT10.S4.Q24 - Rate of inflation and rate of return

juuyuu95juuyuu95 Member

This one is making my head hurt. Can someone help explain?

Comments

  • edited April 2019 1025 karma

    1) If the most profitable stock in the stock market lost 10% of its profit, then all other investments must have, at a minimum, 10% less profit. If your investment in Apple stock declined at a higher %, it must be true that ____

    You lost more profit with Apple than the most profitable stock.

    2) If the poorest person in the world makes $10 million a year, then all other people must make, at a minimum, $10 million a year. If you make more then this value, then it must be true that ________

    You make more than the poorest person.

    3) If the warmest place on Antarctica is -20F, then all other areas must be, at a minimum, colder than -20F. If, where you are standing in Antarctica, the value declines lower than this number, it must be true that ____

    It's colder where you are standing than the warmest place on Antarctica.

    This is a word game. It's hard to see when they are talking about inflation and profits because one must understand that "inflation exceeds the rate of return" simply means they are losing money. If the best investment is losing money, what does that say about everyone else then? Clearly, if the best "X" thing in the world is actually horrible, that would imply that every other "X" thing is at least as horrible as it.

    I did it in one of the examples, but you can switch the wording and make a similarly hard question by changing "best" to "worst" and the value from negative to positive.

    4) When the rate of inflation falls to half the rate of return on the least profitable investment available, the difference between those two rates will be the percentage by which, at a minimum, the value of any investment will increase. If in such a circumstance the value of a particular investment increases by more than that percentage, it must be true that __________

    C) the investment in question is more profitable than the least profitable investment available.

  • BlindReviewerBlindReviewer Alum Member
    855 karma

    The above comment is really helpful, and I just wanted to write it out also because this gave me a headache as well!

    Disclaimer -- I have no background in finance/econ, so while my understanding of what is actually happening here might be wrong, I think it helps break it down more based on grammar than area specific knowledge.

    Let's just call rate of inflation "RI" and rate of return of the most profitable investment "RRMP." When RI is greater than RRMP, then the difference is the percentage by which any other investment will decrease in "value." (This makes sense, because if RRMP is based on the "most profitable" than it should have the smallest difference when RI exceeds RRMP.)

    The last sentence is tricky because of referential phrasing -- it's saying "if in such a circumstance" which means when RI has exceeded RRMP, and then asks what must be true when this happens.

    Before going into the answer choices, it's important to see the cookie cutter structure here of the LSAT presenting a situation that has two possibilities, but never explicitly states what happened. If RI has exceeded RRMP, then either RI has increased, or RRMP has decreased. We don't know what actually happened -- this in itself takes out A and D. E, on the other hand, is just out of scope -- why would there be a change in the most profitable investment? The change in RI vs RRMP doesn't necessarily mean the most profitable investment has changed.

    This leaves us with B and C, and this was really hard. I would just take the guess here, because it's hard to really know without understanding the actual concepts presented. Ultimately, it boils down to an instinct regarding the cookie cutter trap the LSAT uses regarding percentages/rates. I don't necessarily understand how "value" and "profit" differ, but just because it's based on rates, I thought it probably wasn't B. I think that it's wrong because the value of the investment has declined, but it isn't necessarily continuing to decline? It has become less profitable but not necessarily on a downward trend?

    Anyway, hope this helps!

  • juuyuu95juuyuu95 Member
    130 karma

    Oh the examples are very helpful! Thank you both!

Sign In or Register to comment.